Dimension of the root spaces of a semisimple complex Lie algebra












1












$begingroup$


I have problems in understanding the proof that the root spaces of a semisimple Lie algebra are all 1-dimensional and that the only multiples of a root $alpha in Phi$ which occur in $Phi$ are $pm alpha$.



The proof which I am referring to is that of page 101 of the book Introduction to Lie algebras. Here is the web page with the book in Google Books: http://books.google.it/books?id=Kbh3-hvDSbIC&pg=PA97&dq=root+space+decomposition&hl=it&sa=X&ei=ZiAbUqCIAunx4QTlooD4Dw&ved=0CFwQ6AEwBQ#v=onepage&q=root%20space%20decomposition&f=false



The problems begin with the discussion of the two cases $s$ even/odd. I agree with the authors that in the even case there must be an $h_alpha$-eigenvector $v in Vsimeq V_s$ of zero eigenvalue (I think this follows from the fact that $h in mathfrak{sl}_2(mathbb{C})$ acts diagonalizably on $V_s$ with eigenvalues $-s,-s+2,dots, s-2,s$ so $0$ is an eigenvalue because $s$ is even by hypothesis). But how do they deduce from this that $alpha(v)=0$? It seems to me - but it is perhaps a wrong conjecture - that they use the fact that the $0$-eigenspace of the action of $h_alpha$ on $L$ is all contained in the direct sum $K oplus mathfrak{sl}(alpha)$, but I don't understand neither why this fact is true nor how the result can be derived from it.



Can anyone help me, please? Thank you!










share|cite|improve this question









$endgroup$












  • $begingroup$
    Unfortunately I cannot see page 101 in the google preview. A good answer for the statement was given here.
    $endgroup$
    – Torsten Schoeneberg
    Jul 5 '17 at 20:11
















1












$begingroup$


I have problems in understanding the proof that the root spaces of a semisimple Lie algebra are all 1-dimensional and that the only multiples of a root $alpha in Phi$ which occur in $Phi$ are $pm alpha$.



The proof which I am referring to is that of page 101 of the book Introduction to Lie algebras. Here is the web page with the book in Google Books: http://books.google.it/books?id=Kbh3-hvDSbIC&pg=PA97&dq=root+space+decomposition&hl=it&sa=X&ei=ZiAbUqCIAunx4QTlooD4Dw&ved=0CFwQ6AEwBQ#v=onepage&q=root%20space%20decomposition&f=false



The problems begin with the discussion of the two cases $s$ even/odd. I agree with the authors that in the even case there must be an $h_alpha$-eigenvector $v in Vsimeq V_s$ of zero eigenvalue (I think this follows from the fact that $h in mathfrak{sl}_2(mathbb{C})$ acts diagonalizably on $V_s$ with eigenvalues $-s,-s+2,dots, s-2,s$ so $0$ is an eigenvalue because $s$ is even by hypothesis). But how do they deduce from this that $alpha(v)=0$? It seems to me - but it is perhaps a wrong conjecture - that they use the fact that the $0$-eigenspace of the action of $h_alpha$ on $L$ is all contained in the direct sum $K oplus mathfrak{sl}(alpha)$, but I don't understand neither why this fact is true nor how the result can be derived from it.



Can anyone help me, please? Thank you!










share|cite|improve this question









$endgroup$












  • $begingroup$
    Unfortunately I cannot see page 101 in the google preview. A good answer for the statement was given here.
    $endgroup$
    – Torsten Schoeneberg
    Jul 5 '17 at 20:11














1












1








1





$begingroup$


I have problems in understanding the proof that the root spaces of a semisimple Lie algebra are all 1-dimensional and that the only multiples of a root $alpha in Phi$ which occur in $Phi$ are $pm alpha$.



The proof which I am referring to is that of page 101 of the book Introduction to Lie algebras. Here is the web page with the book in Google Books: http://books.google.it/books?id=Kbh3-hvDSbIC&pg=PA97&dq=root+space+decomposition&hl=it&sa=X&ei=ZiAbUqCIAunx4QTlooD4Dw&ved=0CFwQ6AEwBQ#v=onepage&q=root%20space%20decomposition&f=false



The problems begin with the discussion of the two cases $s$ even/odd. I agree with the authors that in the even case there must be an $h_alpha$-eigenvector $v in Vsimeq V_s$ of zero eigenvalue (I think this follows from the fact that $h in mathfrak{sl}_2(mathbb{C})$ acts diagonalizably on $V_s$ with eigenvalues $-s,-s+2,dots, s-2,s$ so $0$ is an eigenvalue because $s$ is even by hypothesis). But how do they deduce from this that $alpha(v)=0$? It seems to me - but it is perhaps a wrong conjecture - that they use the fact that the $0$-eigenspace of the action of $h_alpha$ on $L$ is all contained in the direct sum $K oplus mathfrak{sl}(alpha)$, but I don't understand neither why this fact is true nor how the result can be derived from it.



Can anyone help me, please? Thank you!










share|cite|improve this question









$endgroup$




I have problems in understanding the proof that the root spaces of a semisimple Lie algebra are all 1-dimensional and that the only multiples of a root $alpha in Phi$ which occur in $Phi$ are $pm alpha$.



The proof which I am referring to is that of page 101 of the book Introduction to Lie algebras. Here is the web page with the book in Google Books: http://books.google.it/books?id=Kbh3-hvDSbIC&pg=PA97&dq=root+space+decomposition&hl=it&sa=X&ei=ZiAbUqCIAunx4QTlooD4Dw&ved=0CFwQ6AEwBQ#v=onepage&q=root%20space%20decomposition&f=false



The problems begin with the discussion of the two cases $s$ even/odd. I agree with the authors that in the even case there must be an $h_alpha$-eigenvector $v in Vsimeq V_s$ of zero eigenvalue (I think this follows from the fact that $h in mathfrak{sl}_2(mathbb{C})$ acts diagonalizably on $V_s$ with eigenvalues $-s,-s+2,dots, s-2,s$ so $0$ is an eigenvalue because $s$ is even by hypothesis). But how do they deduce from this that $alpha(v)=0$? It seems to me - but it is perhaps a wrong conjecture - that they use the fact that the $0$-eigenspace of the action of $h_alpha$ on $L$ is all contained in the direct sum $K oplus mathfrak{sl}(alpha)$, but I don't understand neither why this fact is true nor how the result can be derived from it.



Can anyone help me, please? Thank you!







lie-algebras root-systems






share|cite|improve this question













share|cite|improve this question











share|cite|improve this question




share|cite|improve this question










asked Aug 26 '13 at 9:53









DiogenesDiogenes

273110




273110












  • $begingroup$
    Unfortunately I cannot see page 101 in the google preview. A good answer for the statement was given here.
    $endgroup$
    – Torsten Schoeneberg
    Jul 5 '17 at 20:11


















  • $begingroup$
    Unfortunately I cannot see page 101 in the google preview. A good answer for the statement was given here.
    $endgroup$
    – Torsten Schoeneberg
    Jul 5 '17 at 20:11
















$begingroup$
Unfortunately I cannot see page 101 in the google preview. A good answer for the statement was given here.
$endgroup$
– Torsten Schoeneberg
Jul 5 '17 at 20:11




$begingroup$
Unfortunately I cannot see page 101 in the google preview. A good answer for the statement was given here.
$endgroup$
– Torsten Schoeneberg
Jul 5 '17 at 20:11










1 Answer
1






active

oldest

votes


















0












$begingroup$

If $xin M$ is such that $h_{alpha}cdot x = [h_{alpha}, x] = 0$, then $xin Kbigoplusmathfrak{sl}(alpha)$, otherwise $xinmathfrak{g}_{calpha}$ for some $calphainPhi$ and $cneq 0$, but then $[h_{alpha},x]= 2cxneq 0$ giving us the contradiction.



To show that $vin K$, note that $mathfrak{h} = Kbigoplus span({h_{alpha}})$ (exercise), it is also easy to show that $vinmathfrak{h}$. Finally, since $M=Kbigoplus mathfrak{sl}(alpha)bigoplus W$ and $vin Vsubseteq W$, $v$ must lie in $K$.






share|cite|improve this answer











$endgroup$














    Your Answer








    StackExchange.ready(function() {
    var channelOptions = {
    tags: "".split(" "),
    id: "69"
    };
    initTagRenderer("".split(" "), "".split(" "), channelOptions);

    StackExchange.using("externalEditor", function() {
    // Have to fire editor after snippets, if snippets enabled
    if (StackExchange.settings.snippets.snippetsEnabled) {
    StackExchange.using("snippets", function() {
    createEditor();
    });
    }
    else {
    createEditor();
    }
    });

    function createEditor() {
    StackExchange.prepareEditor({
    heartbeatType: 'answer',
    autoActivateHeartbeat: false,
    convertImagesToLinks: true,
    noModals: true,
    showLowRepImageUploadWarning: true,
    reputationToPostImages: 10,
    bindNavPrevention: true,
    postfix: "",
    imageUploader: {
    brandingHtml: "Powered by u003ca class="icon-imgur-white" href="https://imgur.com/"u003eu003c/au003e",
    contentPolicyHtml: "User contributions licensed under u003ca href="https://creativecommons.org/licenses/by-sa/3.0/"u003ecc by-sa 3.0 with attribution requiredu003c/au003e u003ca href="https://stackoverflow.com/legal/content-policy"u003e(content policy)u003c/au003e",
    allowUrls: true
    },
    noCode: true, onDemand: true,
    discardSelector: ".discard-answer"
    ,immediatelyShowMarkdownHelp:true
    });


    }
    });














    draft saved

    draft discarded


















    StackExchange.ready(
    function () {
    StackExchange.openid.initPostLogin('.new-post-login', 'https%3a%2f%2fmath.stackexchange.com%2fquestions%2f476423%2fdimension-of-the-root-spaces-of-a-semisimple-complex-lie-algebra%23new-answer', 'question_page');
    }
    );

    Post as a guest















    Required, but never shown

























    1 Answer
    1






    active

    oldest

    votes








    1 Answer
    1






    active

    oldest

    votes









    active

    oldest

    votes






    active

    oldest

    votes









    0












    $begingroup$

    If $xin M$ is such that $h_{alpha}cdot x = [h_{alpha}, x] = 0$, then $xin Kbigoplusmathfrak{sl}(alpha)$, otherwise $xinmathfrak{g}_{calpha}$ for some $calphainPhi$ and $cneq 0$, but then $[h_{alpha},x]= 2cxneq 0$ giving us the contradiction.



    To show that $vin K$, note that $mathfrak{h} = Kbigoplus span({h_{alpha}})$ (exercise), it is also easy to show that $vinmathfrak{h}$. Finally, since $M=Kbigoplus mathfrak{sl}(alpha)bigoplus W$ and $vin Vsubseteq W$, $v$ must lie in $K$.






    share|cite|improve this answer











    $endgroup$


















      0












      $begingroup$

      If $xin M$ is such that $h_{alpha}cdot x = [h_{alpha}, x] = 0$, then $xin Kbigoplusmathfrak{sl}(alpha)$, otherwise $xinmathfrak{g}_{calpha}$ for some $calphainPhi$ and $cneq 0$, but then $[h_{alpha},x]= 2cxneq 0$ giving us the contradiction.



      To show that $vin K$, note that $mathfrak{h} = Kbigoplus span({h_{alpha}})$ (exercise), it is also easy to show that $vinmathfrak{h}$. Finally, since $M=Kbigoplus mathfrak{sl}(alpha)bigoplus W$ and $vin Vsubseteq W$, $v$ must lie in $K$.






      share|cite|improve this answer











      $endgroup$
















        0












        0








        0





        $begingroup$

        If $xin M$ is such that $h_{alpha}cdot x = [h_{alpha}, x] = 0$, then $xin Kbigoplusmathfrak{sl}(alpha)$, otherwise $xinmathfrak{g}_{calpha}$ for some $calphainPhi$ and $cneq 0$, but then $[h_{alpha},x]= 2cxneq 0$ giving us the contradiction.



        To show that $vin K$, note that $mathfrak{h} = Kbigoplus span({h_{alpha}})$ (exercise), it is also easy to show that $vinmathfrak{h}$. Finally, since $M=Kbigoplus mathfrak{sl}(alpha)bigoplus W$ and $vin Vsubseteq W$, $v$ must lie in $K$.






        share|cite|improve this answer











        $endgroup$



        If $xin M$ is such that $h_{alpha}cdot x = [h_{alpha}, x] = 0$, then $xin Kbigoplusmathfrak{sl}(alpha)$, otherwise $xinmathfrak{g}_{calpha}$ for some $calphainPhi$ and $cneq 0$, but then $[h_{alpha},x]= 2cxneq 0$ giving us the contradiction.



        To show that $vin K$, note that $mathfrak{h} = Kbigoplus span({h_{alpha}})$ (exercise), it is also easy to show that $vinmathfrak{h}$. Finally, since $M=Kbigoplus mathfrak{sl}(alpha)bigoplus W$ and $vin Vsubseteq W$, $v$ must lie in $K$.







        share|cite|improve this answer














        share|cite|improve this answer



        share|cite|improve this answer








        edited Dec 29 '18 at 20:57

























        answered Dec 29 '18 at 16:31









        Sid CarolineSid Caroline

        1,5982515




        1,5982515






























            draft saved

            draft discarded




















































            Thanks for contributing an answer to Mathematics Stack Exchange!


            • Please be sure to answer the question. Provide details and share your research!

            But avoid



            • Asking for help, clarification, or responding to other answers.

            • Making statements based on opinion; back them up with references or personal experience.


            Use MathJax to format equations. MathJax reference.


            To learn more, see our tips on writing great answers.




            draft saved


            draft discarded














            StackExchange.ready(
            function () {
            StackExchange.openid.initPostLogin('.new-post-login', 'https%3a%2f%2fmath.stackexchange.com%2fquestions%2f476423%2fdimension-of-the-root-spaces-of-a-semisimple-complex-lie-algebra%23new-answer', 'question_page');
            }
            );

            Post as a guest















            Required, but never shown





















































            Required, but never shown














            Required, but never shown












            Required, but never shown







            Required, but never shown

































            Required, but never shown














            Required, but never shown












            Required, but never shown







            Required, but never shown







            Popular posts from this blog

            Le Mesnil-Réaume

            Ida-Boy-Ed-Garten

            web3.py web3.isConnected() returns false always